Saltar al contenido principal
LibreTexts Español

8.10: Order-Completitud

  • Page ID
    118353
    • Bob Dumas and John E. McCarthy
    • University of Washington and Washington University in St. Louis
    \( \newcommand{\vecs}[1]{\overset { \scriptstyle \rightharpoonup} {\mathbf{#1}} } \) \( \newcommand{\vecd}[1]{\overset{-\!-\!\rightharpoonup}{\vphantom{a}\smash {#1}}} \)\(\newcommand{\id}{\mathrm{id}}\) \( \newcommand{\Span}{\mathrm{span}}\) \( \newcommand{\kernel}{\mathrm{null}\,}\) \( \newcommand{\range}{\mathrm{range}\,}\) \( \newcommand{\RealPart}{\mathrm{Re}}\) \( \newcommand{\ImaginaryPart}{\mathrm{Im}}\) \( \newcommand{\Argument}{\mathrm{Arg}}\) \( \newcommand{\norm}[1]{\| #1 \|}\) \( \newcommand{\inner}[2]{\langle #1, #2 \rangle}\) \( \newcommand{\Span}{\mathrm{span}}\) \(\newcommand{\id}{\mathrm{id}}\) \( \newcommand{\Span}{\mathrm{span}}\) \( \newcommand{\kernel}{\mathrm{null}\,}\) \( \newcommand{\range}{\mathrm{range}\,}\) \( \newcommand{\RealPart}{\mathrm{Re}}\) \( \newcommand{\ImaginaryPart}{\mathrm{Im}}\) \( \newcommand{\Argument}{\mathrm{Arg}}\) \( \newcommand{\norm}[1]{\| #1 \|}\) \( \newcommand{\inner}[2]{\langle #1, #2 \rangle}\) \( \newcommand{\Span}{\mathrm{span}}\)\(\newcommand{\AA}{\unicode[.8,0]{x212B}}\)

    Damos un argumento a favor de la inccountability de\(\mathbb{R}\) depender únicamente de sus propiedades abstractas de orden.

    DEFINICIÓN. Orden-completo Let\((X, \leq)\) Ser un conjunto ordenado linealmente. Se llama order-complete si, siempre\(A\) y\(B\) son subconjuntos no vacíos de\(X\) con la propiedad que\[(\forall a \in A)(\forall b \in B) \quad a \leq b,\] entonces existe\(c\) en\(X\) tal que\[(\forall a \in A)(\forall b \in B) \quad a \leq c \leq b .\] Tenga en cuenta que cualquier order-complete set debe tener la menor propiedad de límite superior - si\(A\) es cualquier conjunto delimitado no vacío, let\(B\) ser el conjunto de todos los límites superiores para\(A\), y luego\(c\) from (8.20) es el límite superior mínimo (único) para \(A\).

    DEFINICIÓN. Denso Let\((X, \leq)\) Ser un conjunto linealmente ordenado, y\(Y \subseteq\)\(X\). Decimos que\(Y\) es denso en\(X\) si\[(\forall a<b \in X)(\exists y \in Y) a<y<b .\] Definición. Extensión Let\(\left(X, \leq_{X}\right)\) y\(\left(Y, \leq_{Y}\right)\) ser conjuntos ordenados linealmente. Decimos que\(\left(Y, \leq_{Y}\right)\) es una extensión de\(\left(X, \leq_{X}\right)\) si\(X \subseteq Y\) y, para todos\(x_{1}, x_{2}\) en\(X\),\[x_{1} \leq_{X} \quad x_{2} \quad \text { iff } \quad x_{1} \leq_{Y} x_{2} .\] TEOREMA 8.21. Dejar\((X, \leq)\) ser una extensión de\((\mathbb{Q}, \leq)\). Si\((X, \leq)\) es orden-completo y\(\mathbb{Q}\) es denso en\(X\), entonces\(X\) es incontable.

    Prueba. Supongamos que\(X\) es una extensión completa de orden contable\(\mathbb{Q}\) y que\(\mathbb{Q}\) es densa en\(X\).

    Que la secuencia\(\left\langle a_{n} \mid n \in \mathbb{N}\right\rangle\) sea una bijección de\(\mathbb{N}\) a\(X\). Observe que la secuencia impone un orden sobre\(X\). Dejar\(\preceq\) ser definido\(X\) por Es\[(\forall m, n \in \mathbb{N}) a_{m} \preceq a_{n} \Longleftrightarrow m \leq n .\] decir, para cualquiera\(x, y \in X, x \preceq y\) si\(x\) aparece en la secuencia\(\left\langle a_{n}\right\rangle\) anterior\(y\). Entonces\(\preceq\) es un buen orden de\(X\).

    Dado\(Y \subseteq X\) y\(y_{0} \in Y\), decimos que\(y_{0}\) es el elemento\(\preceq\) -minimal de\(Y\) if\[(\forall x \in Y) y_{0} \preceq x .\] Así que cada subconjunto de\(X\) tiene un elemento\(\preceq\) -minimal.

    Vamos a definir dos subsecuencias de\(\left\langle a_{n}\right\rangle\), llamado\(\left\langle a_{f(n)}\right\rangle\) y\(\left\langle a_{g(n)}\right\rangle\), de modo que para cualquier\(n \in \mathbb{N}\)

    (1)\(f(n+1)>g(n)\)

    (2)\(g(n)>f(n)\)

    (3)\(a_{f(n+1)}\) es el elemento\(\preceq\) -minimal del conjunto\[\left\{y \in X \mid a_{f(n)}<y<a_{g(n)}\right\}\] (4)\(a_{g(n+1)}\) es el elemento\(\preceq\) -minimal del conjunto\[\left\{y \in X \mid a_{f(n+1)}<y<a_{g(n)}\right\} .\] Definimos las subsecuencias por recursión usando la secuencia\(\left\langle a_{n}\right\rangle\) para controlar cuidadosamente la construcción. Este argumento se llama argumento de ida y vuelta. Dadas secuencias finitas de longitud que\(N\) satisfacen las propiedades enumeradas anteriormente, definimos\(a_{f(N+1)}\) sujetas a restricciones impuestas por\(a_{f(N)}\) y\(a_{g(N)}\). Luego definimos\(a_{g(N+1)}\) sujetos a las limitaciones impuestas por\(a_{f(N+1)}\) y\(a_{g(N)}\). Luego definimos\(a_{f(N+2)}, a_{g(N+2)}\), y así sucesivamente.

    Vamos\(f(0)=0\). Entonces\(a_{f(0)}=a_{0}\). Dejar\(g(0)\) ser el entero más pequeño\(n\) tal que\(a_{0}<a_{n}\). Tenga en cuenta que esto equivale a definir\(g(0)\) por lo que\(a_{g(0)}\) es el elemento\(\preceq\) -minimal de\(X\) mayor que\(a_{0}\). Supongamos que hemos definido subsecuencias finitas\(\left\langle a_{f(n)} \mid n \leq N\right\rangle,\left\langle a_{g(n)} \mid n \leq N\right\rangle\) que satisfacen las propiedades de orden enumeradas anteriormente. Definiremos\(a_{f(N+1)}\) y\(a_{g(N+1)}\) satisfaceremos las propiedades de pedido enumeradas anteriormente. El conjunto\(X\) contiene los números racionales y dado que\(\mathbb{Q}\) es denso en\(X\), hay un elemento de\(X\),\(x\), tal que\[a_{f(N+1)}<x<g_{(N+1)} .\] Let\(a_{f(N+1)}\) be the\(\preceq\) -elemento mínimo de\(X\) tal que\[a_{f(N)}<a_{f(N+1)}<a_{g(N)} .\] Since\(\preceq\) es un orden bien definido de\(X, f(N+1)\) está bien definido. Dejamos\(a_{g(N+1)}\) ser el elemento\(\preceq\) -minimal de\(X\) tal que\[a_{f(N+1)}<a_{g(N+1)}<a_{g(N)} .\] Por nuestra discusión anterior,\(g(N+1)\) está bien definido. Observe que para cualquier\(m, n \in \mathbb{N}\),\[a_{f(m)}<a_{g(n)} .\] Por lo tanto la secuencia creciente\(\left\langle a_{f(n)} \mid n \in \mathbb{N}\right\rangle\) está delimitada arriba, y por Lema 8.5, la secuencia converge a su límite inferior superior,\(a\).

    Para cualquiera\(n \in \mathbb{N}\),\[a_{f(n)}<a<a_{g(n) .} .\] So no\(a\) es un término de ninguna subsecuencia. Demostramos que no\(a\) es un término en la secuencia\(\left\langle a_{n}\right\rangle\). Supongamos a modo de contradicción que\(a=a_{n}\) para algunos\(n \in \mathbb{N}\). Ya que\(f(0)=0, n \neq 0\). Let\[Y=(f[\mathbb{N}] \cup g[\mathbb{N}]) \cap\ulcorner n\urcorner .\] Entonces\(Y \neq \emptyset\) es finito, y tiene un elemento máximo.

    Si el elemento máximo de\(Y\) es\(f(0)\), entonces para cada\(1 \leq k<n\), debemos tener\(a_{k}<a_{0}\). Pero entonces\(g(0)\) sería\(n\), lo que contradice el hecho de que no\(n\) está en el rango de\(g\).

    Si el elemento máximo de\(Y\) es\(f(m+1)\) para algunos\(m\), entonces\(g(m+1)>\)\(n\), y\[f(m+1)<n<g(m+1) .\] Sin embargo\[a_{f(m+1)}<a_{n}<a_{g(m+1)}<a_{g(m) .} .\] Esto es imposible ya que\(a_{g(m+1)}\) es el\(\preceq\) - elemento mínimo de\(X\) en el intervalo abierto\(\left(a_{f(m+1)}, a_{g(m)}\right)\).

    Si el elemento máximo de\(Y\) es\(g(m)\) para algunos\(m\), entonces\(f(m+1)>n\) y\[g(m)<n<f(m+1) .\] Sin embargo\[a_{f(m)}<a_{f(m+1)}<a_{n}<a_{g(m)} .\] Esto es imposible ya que\(a_{f(m+1)}\) es el elemento\(\preceq\) -minimal de \(X\)en el intervalo abierto\(\left(a_{f(m)}, a_{g(m)}\right)\). Entonces no\(a\) es un término en la secuencia\(\left\langle a_{n}\right\rangle\). Por lo tanto no hay bijección de\(\mathbb{N}\) a\(X\), y\(X\) es incontable.

    Por el Ejercicio 8.20,\(\mathbb{Q}\) es denso en\(\mathbb{R}\). Como el conjunto de números reales está ordenado completo por el teorema de límite superior mínimo, obtenemos:

    COROLARIO 8.22. El conjunto de números reales es incontable.

    TEOREMA 8.23. Dejar\(\left(X, \leq_{X}\right)\) ser una extensión orden-completa de\(\mathbb{Q}\) en la que\(\mathbb{Q}\) es densa, y tal que no\(X\) tiene elemento máximo o mínimo. Luego hay una bijección conservadora de orden de\(\mathbb{R}\) hacia adelante\(X\) que es la identidad en adelante\(\mathbb{Q}\).

    Prueba. Definamos un mapa\(f: \mathbb{R} \rightarrow X\). Si\(q \in \mathbb{Q}\), defina\(f(q)=q\). Si\(\alpha \in \mathbb{R} \backslash \mathbb{Q}\), defina\(f(\alpha)\) que sea el límite inferior superior en\(X\) de\(\{q \in \mathbb{Q} \mid q \leq \alpha\}\). La función\(f\) está bien definida, porque\(X\) tiene la propiedad Lost Upper Bound. Es inyectivo, porque si\(\alpha \neq \beta\), hay números racionales entre\(\alpha\) y\(\beta\).

    Mostrar\(f\) es sobre, supongamos\(x \in X\). \(\alpha \in \mathbb{R}\)Definir para ser el límite inferior superior en\(\mathbb{R}\) de\(\left\{q \in \mathbb{Q} \mid q \leq_{X} x\right\}\). Entonces\(f(\alpha)=x\).

    Finalmente,\(f\) se preserva el orden porque si\(\alpha \leq \beta\), entonces\(f(\beta)\) se define como el límite inferior superior de un superconjunto del conjunto cuyo límite inferior superior es\(f(\alpha)\), y así\(f(\alpha) \leq_{X} f(\beta)\).

    OBSERVACIÓN. ¿Qué pasa si bajamos el requisito que no\(X\) tienen elemento máximo o mínimo?


    This page titled 8.10: Order-Completitud is shared under a CC BY 4.0 license and was authored, remixed, and/or curated by Bob Dumas and John E. McCarthy via source content that was edited to the style and standards of the LibreTexts platform; a detailed edit history is available upon request.